GMAT Prep | When a city experiences a sharp decline in popul

This topic has expert replies
User avatar
Legendary Member
Posts: 698
Joined: Tue Jul 21, 2015 12:12 am
Location: Noida, India
Thanked: 32 times
Followed by:26 members
GMAT Score:740
When a city experiences a sharp decline in population, the city's tax revenues, which pay for such city services as police protection and maintenance of water lines, also decrease. The area to be policed and the number and length of the water lines to be maintained, however, do not decrease. Attempting to make up the tax revenue lost by raising tax rates is not feasible, since higher tax rates would cause even more residents to leave.

The information given most strongly supports which of the following general claims?

(A) If, in a city with sharply declining population, police protection and water line maintenance do not deteriorate, some other service previously provided by the city will deteriorate or be eliminated.

(B) If a city's tax rates are held stable over a period of time, neither the population nor the levels of city services provided will tend to decline over that period.

(C) If a city's population declines sharply, police protection and water line maintenance are the services that deteriorate most immediately and most markedly.

(D) A city that suffers revenue losses because of a sharp decline in population can make up some of the lost tax revenue by raising tax rates, provided the city's tax rates are low in relation to those other cities.

(E) A city that is losing residents because tax rates are perceived as too high by those residents can reverse this population trend by bringing its tax rates down to a more moderate level.
R I C H A,
My GMAT Journey: 470 → 720 → 740
Target Score: 760+
[email protected]
1. Press thanks if you like my solution.
2. Contact me if you are not improving. (No Free Lunch!)

User avatar
Legendary Member
Posts: 698
Joined: Tue Jul 21, 2015 12:12 am
Location: Noida, India
Thanked: 32 times
Followed by:26 members
GMAT Score:740

by richachampion » Thu Oct 13, 2016 10:04 am
OA: A

I think we do not have a well defined conclusion in this question. what say experts?

Or this is the conclusion -
Attempting to make up the tax revenue lost by raising tax rates is not feasible

I will love to discuss Option B in detail as this option is testing a very powerful concept of causality.
R I C H A,
My GMAT Journey: 470 → 720 → 740
Target Score: 760+
[email protected]
1. Press thanks if you like my solution.
2. Contact me if you are not improving. (No Free Lunch!)

User avatar
GMAT Instructor
Posts: 15539
Joined: Tue May 25, 2010 12:04 pm
Location: New York, NY
Thanked: 13060 times
Followed by:1906 members
GMAT Score:790

by GMATGuruNY » Sat Oct 15, 2016 3:53 am
richachampion wrote:OA: A

I think we do not have a well defined conclusion in this question. what say experts?
The information given most strongly supports which of the following general claims?
Here, the passage must support the correct answer choice.
Implication:
The correct answer is an INFERENCE, a statement that must be true, given the information in the passage.
Generally, the passage for an inference CR will not have a conclusion.
Or this is the conclusion -
Attempting to make up the tax revenue lost by raising tax rates is not feasible
This information is not a conclusion but a FACT.
I will love to discuss Option B in detail as this option is testing a very powerful concept of causality.
One approach is to apply the NEGATION TEST.
When the correct answer choice is negated, the passage will be contradicted.
B, negated:
If a city's tax rates are held stable over a period of time, the population or the levels of city services provided will tend to decline over that period.
The passage offers no information about a city in which tax rates are held stable.
Since the negation of B does not contradict the passage, eliminate B.

A, negated:
If, in a city with sharply declining population, police protection and water line maintenance do not deteriorate, no other service previously provided by the city will deteriorate or be eliminated.
This negation contradicts the passage's contention that a city with a sharply declining population will have less tax revenue to pay for city services.
Since the negation of A contradicts the passage, A is the correct inference: the answer choice that MUST BE TRUE, given the information in the passage.

The correct answer is A.
Private tutor exclusively for the GMAT and GRE, with over 20 years of experience.
Followed here and elsewhere by over 1900 test-takers.
I have worked with students based in the US, Australia, Taiwan, China, Tajikistan, Kuwait, Saudi Arabia -- a long list of countries.
My students have been admitted to HBS, CBS, Tuck, Yale, Stern, Fuqua -- a long list of top programs.

As a tutor, I don't simply teach you how I would approach problems.
I unlock the best way for YOU to solve problems.

For more information, please email me (Mitch Hunt) at [email protected].
Student Review #1
Student Review #2
Student Review #3

Legendary Member
Posts: 944
Joined: Wed May 30, 2012 8:21 am
Thanked: 8 times
Followed by:5 members

by RBBmba@2014 » Thu Oct 27, 2016 4:45 am
Hi Verbal Experts(Ceilidh/Dave/Mitch/Marty/others),
Can you please shed some light on the following concerns:

1. Why C is wrong ?

2. Option D is wrong because it appears to CONTRADICT the PREMISE (Attempting to make up the tax revenue lost by raising tax rates is not feasible) in the ARGUMENT. A PREMISE is a undisputed FACT, so it can't be contradicted. Right ?
Last edited by RBBmba@2014 on Mon Dec 26, 2016 10:58 pm, edited 1 time in total.

Legendary Member
Posts: 944
Joined: Wed May 30, 2012 8:21 am
Thanked: 8 times
Followed by:5 members

by RBBmba@2014 » Mon Dec 26, 2016 8:48 pm
Hi Verbal Experts,
Any thoughts on my above concerns ?

Look forward to hear from you! Much thanks in advance.

User avatar
Legendary Member
Posts: 2663
Joined: Wed Jan 14, 2015 8:25 am
Location: Boston, MA
Thanked: 1153 times
Followed by:128 members
GMAT Score:770

by DavidG@VeritasPrep » Sat Dec 31, 2016 11:22 am
RBBmba@2014 wrote:Hi Verbal Experts(Ceilidh/Dave/Mitch/Marty/others),
Can you please shed some light on the following concerns:

1. Why C is wrong ?

2. Option D is wrong because it appears to CONTRADICT the PREMISE (Attempting to make up the tax revenue lost by raising tax rates is not feasible) in the ARGUMENT. A PREMISE is a undisputed FACT, so it can't be contradicted. Right ?
1) Imagine you're in charge of setting the budget for the city. You're told that because the city's population is down, tax revenue is down. You also know that the costs of police protection and water line maintenance are unchanged. So you know you have to cut spending somewhere, but it's hardly a forgone conclusion that the cuts will most profoundly impact police protection and water line maintenance. You could make cuts there. But you may also decide that those things are pretty important and that some other budget item is a lower priority and cut spending there.

2) Correct.
Veritas Prep | GMAT Instructor

Veritas Prep Reviews
Save $100 off any live Veritas Prep GMAT Course

Legendary Member
Posts: 944
Joined: Wed May 30, 2012 8:21 am
Thanked: 8 times
Followed by:5 members

by RBBmba@2014 » Tue Jan 03, 2017 5:26 am
DavidG@VeritasPrep wrote:
RBBmba@2014 wrote: 1. Why C is wrong ?
1) Imagine you're in charge of setting the budget for the city. You're told that because the city's population is down, tax revenue is down. You also know that the costs of police protection and water line maintenance are unchanged. So you know you have to cut spending somewhere, but it's hardly a forgone conclusion that the cuts will most profoundly impact police protection and water line maintenance. You could make cuts there. But you may also decide that those things are pretty important and that some other budget item is a lower priority and cut spending there.
Hi Dave,
I hear you here. Apart from what you've mentioned, I guess, the following could also be a potential reason to eliminate Option C:

"most immediately and most markedly" -- These phrases in C indicate some sort of EXTREMES. GENERALLY, such things should be considered INCORRECT in an INFERENCE CR because the OA of an INFERENCE CR is a MUST BE TRUE statement.

Am I correct ?
Last edited by RBBmba@2014 on Wed Jan 04, 2017 6:01 am, edited 1 time in total.

User avatar
Legendary Member
Posts: 2663
Joined: Wed Jan 14, 2015 8:25 am
Location: Boston, MA
Thanked: 1153 times
Followed by:128 members
GMAT Score:770

by DavidG@VeritasPrep » Tue Jan 03, 2017 9:55 am
RBBmba@2014 wrote:
DavidG@VeritasPrep wrote:
RBBmba@2014 wrote: 1. Why C is wrong ?
1) Imagine you're in charge of setting the budget for the city. You're told that because the city's population is down, tax revenue is down. You also know that the costs of police protection and water line maintenance are unchanged. So you know you have to cut spending somewhere, but it's hardly a forgone conclusion that the cuts will most profoundly impact police protection and water line maintenance. You could make cuts there. But you may also decide that those things are pretty important and that some other budget item is a lower priority and cut spending there.
Hi Dave,
I hear you here. Apart from what you've mentioned, I guess, the following could also be a potential reason to eliminate Option C:

"most immediately and most markedly" -- These phrases in C indicate some sort of EXTREMES. Such things should be considered INCORRECT in an INFERENCE CR because the OA of an INFERENCE CR is a MUST BE TRUE statement.

Am I correct ?
I'd modify that statement a little bit. When you see extreme language in an answer choice for an inference question, it can be considered a reason to be skeptical of the answer, but it doesn't definitively rule the choice out. It's just a higher bar to clear. For example, if an answer choice claims that something is "always" true, and you suspect that it's merely true "most" of the time, that answer choice would be incorrect. But it's not inconceivable that some scenario would always be true, depending on the prompt.
Veritas Prep | GMAT Instructor

Veritas Prep Reviews
Save $100 off any live Veritas Prep GMAT Course

Legendary Member
Posts: 944
Joined: Wed May 30, 2012 8:21 am
Thanked: 8 times
Followed by:5 members

by RBBmba@2014 » Wed Jan 04, 2017 6:06 am
DavidG@VeritasPrep wrote:I'd modify that statement a little bit. When you see extreme language in an answer choice for an inference question, it can be considered a reason to be skeptical of the answer, but it doesn't definitively rule the choice out. It's just a higher bar to clear. For example, if an answer choice claims that something is "always" true, and you suspect that it's merely true "most" of the time, that answer choice would be incorrect. But it's not inconceivable that some scenario would always be true, depending on the prompt.
Yes, of course :-)

Well, I've tweaked my post a BIT with the insertion of "GENERALLY" - Now I think, it makes perfect sense ?

User avatar
Legendary Member
Posts: 2663
Joined: Wed Jan 14, 2015 8:25 am
Location: Boston, MA
Thanked: 1153 times
Followed by:128 members
GMAT Score:770

by DavidG@VeritasPrep » Wed Jan 04, 2017 9:40 am
RBBmba@2014 wrote:
DavidG@VeritasPrep wrote:I'd modify that statement a little bit. When you see extreme language in an answer choice for an inference question, it can be considered a reason to be skeptical of the answer, but it doesn't definitively rule the choice out. It's just a higher bar to clear. For example, if an answer choice claims that something is "always" true, and you suspect that it's merely true "most" of the time, that answer choice would be incorrect. But it's not inconceivable that some scenario would always be true, depending on the prompt.
Yes, of course :-)

Well, I've tweaked my post a BIT with the insertion of "GENERALLY" - Now I think, it makes perfect sense ?
Looks good to me :)
Veritas Prep | GMAT Instructor

Veritas Prep Reviews
Save $100 off any live Veritas Prep GMAT Course